6
$\begingroup$

On a four-manifold, there is apparently a relation between the first Pontryagin class modulo 4 and the Pontryagin square of the second Stiefel-Whitney class:

$\mathfrak{P}(w_2) = p_1 \; {\rm mod} \; 4$

This fact is for instance mentioned in the comments of this question, but I have been unable to find a proof of it.

My question is: Is it true that on an 8-manifold, the analogous relation

$\mathfrak{P}(w_4) \stackrel{?}{=} p_2 \; {\rm mod} \; 4$

holds?

$\endgroup$

2 Answers 2

6
$\begingroup$

In your first claim, it is a bit unclear what bundle you are considering. It is false for the tangent bundle of $\mathbb{C}P^2$: $\mathfrak{P}(w_2) = c_1^2 = 9 \; {\rm mod} \; 4$, while $p_1 = 3$. The context of the question you link to is arbitrary oriented rank 3 bundles over a 4-dimensional base. Is that what you have in mind?

The Pontrjagin squares in the cohomology of $BO$ were first computed by Wu in On Pontrjagin classes. III (MR0115179). It is not so easy to get hold of a digital copy of the paper or its translation, but one of the main results is Theorem 5 of Section 4:

$$\mathfrak{P}(w_{2i+1}) = \beta_4 Sq^{2i}w_{2i+1} + \theta_2(w_1 Sq^{2i}w_{2i+1})$$ $$\mathfrak{P}(w_{2i}) = \rho_4 p_i + \beta_4(w_{2i-1}w_{2i}) + \theta_2 \left(w_1 Sq^{2i-1}w_{2i} + \sum_{j=0}^{i - 1} w_{2j}w_{4i-2j}\right)$$ where $\rho_4$ is mod 4 reduction of coefficients, $\beta_4$ is the mod 4 reduction of the Bockstein map $H^i(-;Z/2) \to H^{i+1}(-; Z)$, and $\theta_2 : H^i(-;Z/2) \to H^i(-;Z/4)$ is induced by the inclusion $Z/2 \to Z/4, x \mapsto 2x$.

As a special case, we find that for any orientable vector bundle (i.e. $w_1 = 0$) $$ \mathfrak{P}(w_2) = \rho_4 p_1 + \theta_2(w_4) . $$

It's worth pointing out the corollary that Wu deduces from the above calculation (and that Stiefel-Whitney classes are homotopy invariants by Wu's formula): the mod 4 reductions of the Pontrjagin classes of a closed smooth manifold are homotopy invariants.

$\endgroup$
2
  • $\begingroup$ Yes, indeed it was a bit unclear to me what hypotheses were necessary for the formula I wrote to be valid. The general formula of Wu solves my problem. Thanks! $\endgroup$ May 16, 2014 at 10:09
  • 2
    $\begingroup$ Maybe just a word of warning. The formula above is not quite the same as the formula in my answer below, which can be found in the paper of Emery Thomas. Thomas has a footnote mentioning that his formula differs from the one of Wu because Wu has a different definition of the Pontryagin classes. I don't have access to the paper of Wu so I can't check what this means exactly. All I can say is that Thomas's definition of the Pontryagin classes look like the usual one. $\endgroup$ May 17, 2014 at 14:18
5
$\begingroup$

Ok, a more general statement holds. It is apparently due to Wu, but I found it in this paper by Emery Thomas, Theorem C.

Let $B$ be a vector bundle over a manifold $X$, $w_i$ be its Stiefel-Whitney classes and $p_i$ its Pontryagin classes. Let $\rho_4$ be the reduction modulo 4 and $\theta_2$ be the embedding of $\mathbb{Z}_2$ into $\mathbb{Z}_4$ (as well as their induced actions on cohomology groups). Then

$\mathfrak{P}(w_{2i}) = \rho_4(p_i) + \theta_2 \left( w_1 Sq^{2i-1} w_{2i} + \sum_{j = 0}^{i-1} w_{2j} w_{4i-2j} \right)$.

There is also a formula for the Pontryagin square of odd Stiefel-Whitney classes in the paper.

$\endgroup$

Your Answer

By clicking “Post Your Answer”, you agree to our terms of service and acknowledge you have read our privacy policy.

Not the answer you're looking for? Browse other questions tagged or ask your own question.